Explicit relations between pair correlation of zeros and primes in short intervals (Q442501): Difference between revisions

From MaRDI portal
Added link to MaRDI item.
Import240304020342 (talk | contribs)
Set profile property.
Property / MaRDI profile type
 
Property / MaRDI profile type: MaRDI publication profile / rank
 
Normal rank

Revision as of 01:17, 5 March 2024

scientific article
Language Label Description Also known as
English
Explicit relations between pair correlation of zeros and primes in short intervals
scientific article

    Statements

    Explicit relations between pair correlation of zeros and primes in short intervals (English)
    0 references
    0 references
    0 references
    0 references
    1 August 2012
    0 references
    Let \(\zeta(s)\) denote the Riemann zeta-function, and let \(\rho=\beta+i\gamma\) denote a nontrivial zero of \(\zeta(s)\). Montgomery's pair correlation function is defined as \[ F(X,T) = \sum_{0<\gamma, \gamma'\leq T} X^{i(\gamma-\gamma')} w(\gamma-\gamma'), \quad w(u)=\frac{4}{4+u^2} \] where the sum runs over two sets of ordinates of zeros of \(\zeta(s)\). The paper under review explores the equivalence between the asymptotic behavior of the function \(F(X,T)\) and the distribution of primes in short intervals extending and improving results of \textit{T. H. Chan} [J. Lond. Math. Soc. (2) 68 (2003) 579--598; Zbl 1054.11047]. To state the main results, write \[ F(X,T) = \frac{T}{2\pi} \left( \log \frac{T}{2\pi} - 1 \right) + R_F(X,T) \] and write \[ \int_X^{2X} \left( \psi(x+\theta x)-\psi(x)-\theta x \right)^2 dx = \frac{3}{2} X^2 \theta \left( \log(1/\theta)+C\right) + R_J(X,\theta), \] where \(\psi(x)=\sum_{n\leq x} \Lambda(n)\) is the usual summatory function of von Mangoldt's function \(\Lambda(n)\) and \(C=1-\gamma_0-\log 2\pi\) (\(\gamma_0\) is Euler's constant). Assuming the Riemann hypothesis (RH), the authors investigate the relationship between the error terms \(R_F(X,T)\) and \(R_J(X,\theta)\) with \(X, T,\) and \(\theta\) in suitable ranges. They prove: 1. Assume RH and let \(0<\alpha <1\), \(\beta\geq 0\), and \(1\leq A_1 \leq A_2\). If \[ R_F(X,T) \ll T^{1-\alpha} \log^{-\beta}T \] uniformly for \(X^{1/A_2}/\log^{(3+\beta)}X\leq T \leq X^{1/A_1}/\log^{(3+\beta)}X\), then \[ R_J(X,\theta) \ll X^2 \theta^{1+\alpha/2} \log^{(1-\beta)/2}(1/\theta) \] uniformly for \(X^{-1/A_1(1+\alpha)} \leq \theta \leq X^{-1/A_2(1-\alpha)}\). 2. Assume RH and let \(0<\alpha <1\), \(\beta\geq 0\), \((\alpha,\beta)\neq (0,0)\), and \(1\leq A_1 \leq A_2\). If \[ R_J(X,\theta) \ll X^2 \theta^{1+\alpha} \log^{-\beta}(1/\theta) \] uniformly for \(X^{-(\alpha/2+1)/A_1}/\log^{(3+\beta)}X \leq \theta \leq X^{-1/A_2}/\log^{4(\beta+2)/3}X\), then \[ R_F(X,T) \ll T^{1-\alpha/(\alpha+3)} (\log T)^{(\alpha-\beta+2)/(\alpha+3)} \] uniformly for \(X^{(\alpha+3)/3A_2(1-\alpha)} \leq T \leq X^{(\alpha+3)/3A_1}\). The authors also discuss the relationship between these results and the integral \[ \int_X^{2X} \left( \psi(x+h)-\psi(x)-h \right)^2 dx \] which they note is perhaps the ``more natural'' way to study primes in short intervals.
    0 references
    0 references
    primes in short intervals
    0 references
    Riemann zeta-function
    0 references
    pair correlation of zeros
    0 references